PT58 - Couple Questions Forum

Prepare for the LSAT or discuss it with others in this forum.
Post Reply
ConsideringLawSchool

Bronze
Posts: 313
Joined: Fri Apr 24, 2009 12:18 pm

PT58 - Couple Questions

Post by ConsideringLawSchool » Wed Jan 20, 2010 11:36 pm

A few from September that puzzle me:

Sec 1 (LR) - #23
To me, it seems that the stem offers multiple justifications for imposing the tariff. The justifications include keeping domestic fruit growers in business AND preserving a "unique way of life." I don't understand why the correct answer addresses states that "social concerns" take priority over "economic efficiency" -- isn't keeping the farmers in business also a key consideration (as captured in Answer D)

Sec 1 (LR) - #24 (Bears)
We know that the road closed and that, over the 8 year period thereafter, the population of bears within the preserve (one region of the valley) nearly doubled. I do not see why there is any reason to predict continued growth. The area may well have reached capacity. Perhaps all the growth was in the first couple years. The population wouldn't expand forever, so I really don't see why the population of the preserve or valley should increase. What am I missing here?

Sec 2 (RC) - #11
Throughout this passage, I thought that these parallel computer systems are a type of supercomputer. This belief was supported by the statement that "until that time, supercomputers had been used..." Therefore, B would have to say "supercomputers WERE unable to accurately predict" since now they are able to accurately predict... How am I misunderstanding?

Thanks :-)

Woozy

Bronze
Posts: 159
Joined: Sun Oct 18, 2009 2:29 pm

Re: PT58 - Couple Questions

Post by Woozy » Thu Jan 21, 2010 1:13 am

#23 - Keeping the growers in business is referred to later as a "unique way of life," which highlights its status as a social concern more than an economic one. Also, the mention that there are "more lucrative industrial uses" for the land highlights the fact that the politician's proposal is not done for economic reasons. D is no good because we don't know whether it will be a net positive for the country's own citizens - it will help the fruit growers but will harm domestic fruit consumers.

This question is one of many examples that highlights how an economic background and thought process are assets during the LSAT :wink:

#24 - you're right, it shouldn't, necessarily. Your task is to find an answer choice that will make that more clear.

It is sort of a trick question. They are trying to make you confuse the preserve with the valley itself. If you pay attention you will see that the evidence is that the population of the preserve has increased and the conclusion is that the population of the valley will continue to increase. A quick scope shift they hope you will miss. TCR is that the valley population has actually not been increasing. If it is established that the population of the valley has not been increasing over the last 8 years, clearly the argument that the population will continue to increase is destroyed.

#11 - The passage does not refer to parallel computers as a type of supercomputer, rather as a competing method of computing. You should be reading "until that time supercomputers had been used..." as implying that after that time parallel computers replaced supercomputers in this field. I do not see any other place in the passage that implies parallel computers are a supercomputer.

Sometimes the best way to get these types of tough questions is by eliminating the other choices. If you are stuck on the logic of the right choice, at least make sure you see why the one you picked was clearly wrong.

ConsideringLawSchool

Bronze
Posts: 313
Joined: Fri Apr 24, 2009 12:18 pm

Re: PT58 - Couple Questions

Post by ConsideringLawSchool » Thu Jan 21, 2010 11:46 pm

Woozy wrote:#23 - Keeping the growers in business is referred to later as a "unique way of life," which highlights its status as a social concern more than an economic one. Also, the mention that there are "more lucrative industrial uses" for the land highlights the fact that the politician's proposal is not done for economic reasons. D is no good because we don't know whether it will be a net positive for the country's own citizens - it will help the fruit growers but will harm domestic fruit consumers.

This question is one of many examples that highlights how an economic background and thought process are assets during the LSAT :wink:

#24 - you're right, it shouldn't, necessarily. Your task is to find an answer choice that will make that more clear.

It is sort of a trick question. They are trying to make you confuse the preserve with the valley itself. If you pay attention you will see that the evidence is that the population of the preserve has increased and the conclusion is that the population of the valley will continue to increase. A quick scope shift they hope you will miss. TCR is that the valley population has actually not been increasing. If it is established that the population of the valley has not been increasing over the last 8 years, clearly the argument that the population will continue to increase is destroyed.

#11 - The passage does not refer to parallel computers as a type of supercomputer, rather as a competing method of computing. You should be reading "until that time supercomputers had been used..." as implying that after that time parallel computers replaced supercomputers in this field. I do not see any other place in the passage that implies parallel computers are a supercomputer.

Sometimes the best way to get these types of tough questions is by eliminating the other choices. If you are stuck on the logic of the right choice, at least make sure you see why the one you picked was clearly wrong.
Woozy--you really are the best. Thanks so much for your help...

On the bears, I did catch the scope shift but still the question seems odd to me. Even if it said "thus, the preserve's bear population will increase," I wouldn't see the logic. I suppose there may be slight reason to believe that something that has increased over the past 8 years will continue to increase--except for the fact that the increase was triggered by a change that happened long enough ego such that all the effects already would be evident.

I think that I'm finally starting to make fewer stupid errors (got -0 on both LRs and the LG tonight on PT48). I recently discovered Cambridge LSAT's collection of the 410 hardest LRs from PT 1-38, and those are very helpful. Just trying to figure out what's most useful to do for the final couple weeks :-)

Woozy

Bronze
Posts: 159
Joined: Sun Oct 18, 2009 2:29 pm

Re: PT58 - Couple Questions

Post by Woozy » Fri Jan 22, 2010 2:02 am

ConsideringLawSchool wrote:On the bears, I did catch the scope shift but still the question seems odd to me. Even if it said "thus, the preserve's bear population will increase," I wouldn't see the logic. I suppose there may be slight reason to believe that something that has increased over the past 8 years will continue to increase--except for the fact that the increase was triggered by a change that happened long enough ego such that all the effects already would be evident.

I think that I'm finally starting to make fewer stupid errors (got -0 on both LRs and the LG tonight on PT48). I recently discovered Cambridge LSAT's collection of the 410 hardest LRs from PT 1-38, and those are very helpful. Just trying to figure out what's most useful to do for the final couple weeks :-)
Glad to hear you are making progress. When are you planning to take the test?

For the bears, you know that the argument has a flaw when they ask for what weakens it. The fact that they are asking the question means the argument is open to at least one set of circumstances which will invalidate it or at least call it into question. So, stop trying to fight the logic that we already know must be broken. That gets you no points. What gets you points is finding the one choice out of five that would most damage the argument, flawed though it may already be.

ConsideringLawSchool

Bronze
Posts: 313
Joined: Fri Apr 24, 2009 12:18 pm

Re: PT58 - Couple Questions

Post by ConsideringLawSchool » Fri Jan 22, 2010 2:36 am

Woozy wrote:
ConsideringLawSchool wrote:On the bears, I did catch the scope shift but still the question seems odd to me. Even if it said "thus, the preserve's bear population will increase," I wouldn't see the logic. I suppose there may be slight reason to believe that something that has increased over the past 8 years will continue to increase--except for the fact that the increase was triggered by a change that happened long enough ego such that all the effects already would be evident.

I think that I'm finally starting to make fewer stupid errors (got -0 on both LRs and the LG tonight on PT48). I recently discovered Cambridge LSAT's collection of the 410 hardest LRs from PT 1-38, and those are very helpful. Just trying to figure out what's most useful to do for the final couple weeks :-)
Glad to hear you are making progress. When are you planning to take the test?

For the bears, you know that the argument has a flaw when they ask for what weakens it. The fact that they are asking the question means the argument is open to at least one set of circumstances which will invalidate it or at least call it into question. So, stop trying to fight the logic that we already know must be broken. That gets you no points. What gets you points is finding the one choice out of five that would most damage the argument, flawed though it may already be.
Feb

And yes, you're totally right.

Want to continue reading?

Register now to search topics and post comments!

Absolutely FREE!


JJDancer

Gold
Posts: 1564
Joined: Sun Jul 26, 2009 7:41 pm

Re: PT58 - Couple Questions

Post by JJDancer » Fri Oct 08, 2010 11:53 am

I can't believe I am posting about THE bear question the day before the LSAT buuut..

Can someone explain why D is incorrect?
Doesn't D imply that pop in preserve nearly doubled but that doesn't mean pop of VALLEY increased because pop outside preserve decreased...meaning they migrated to the preserve.

Or would this NOT weaken because that still says the valley overall could increase (just that it was happening in the preserve area while the nonpreserve area decreased but valleyTOTAL increased?)

Woozy

Bronze
Posts: 159
Joined: Sun Oct 18, 2009 2:29 pm

Re: PT58 - Couple Questions

Post by Woozy » Fri Oct 08, 2010 11:27 pm

JJDancer wrote:I can't believe I am posting about THE bear question the day before the LSAT buuut..

Can someone explain why D is incorrect?
Doesn't D imply that pop in preserve nearly doubled but that doesn't mean pop of VALLEY increased because pop outside preserve decreased...meaning they migrated to the preserve.

Or would this NOT weaken because that still says the valley overall could increase (just that it was happening in the preserve area while the nonpreserve area decreased but valleyTOTAL increased?)
Be careful when you think something is implied, one of the main tricks of the LSAC is making you assume more than is written. Within the valley there are two areas: preserve and non-preserve. The population of the preserve (which contains most of the valley's bears) has nearly doubled in the past 8 years. Now answer D tells us the population outside the preserve has decreased. That's just vague enough that it tells you nothing about what has happened to the population of the valley. These sets of facts could be true whether the valley's population as a whole has decreased, increased, or remained the same.

The argument the author is making is that the valley's population will increase. Choice E is the only one that tells you anything definite about what happened to the valley's population over the last 8 years.

Want to continue reading?

Register for access!

Did I mention it was FREE ?


Post Reply

Return to “LSAT Prep and Discussion Forum”